If Koguchi is assigned to area 2, then which one of the following could be true?

on December 14, 2019

Please Explain

Please explain why answer E is correct please?

Reply
Create a free account to read and take part in forum discussions.

Already have an account? log in

Skylar on December 14, 2019

@sporzes thanks for your question.

Based on Rule 1, we know that M is in area 3. Based on this question, we know that we need to put K in area 2. This gives us:
1:
2: K
3: M

Now, the game tells us that each area has at least on ranger, but Rule 2 tells us that neither O nor P can go in area 1. We already have M and K placed outside of area 1, so this only leaves J or L as options for area 1. However, Rule 3 says that L has to go with either K or M, which are both already placed outside of area 1. Therefore L cannot go in area 1, so J is the only option left to fill that spot. This gives us:
1: J
2: K
3: M

Rule 4 tells us that J and K must be placed together if O is in area 2 and that if J and K are not together, O cannot be in area 2. J and K are already placed and are not/cannot be together in the same area, so O cannot be in area two. Rule 2 says that O cannot be in area 1, so O must be assigned to area 3. This gives us:
1: J
2: K
3: MO

Now, let's evaluate each answer choice.
(A) is incorrect because we have concluded that J must be in area 1.
(B) is incorrect for the same reason that (A) is.
(C) is incorrect because, even though we didn't place L in our scenario above, Rule 3 tells us that it has to go with either K or M. Since neither K nor M is in area 1, L cannot go in area 1.
(D) is incorrect because we have concluded that O must be in area 3.

This leaves us with (E), which could be true without violating the rules. This scenario would look like this:
1: J
2: KL
3: MOP

Does that make sense? Please reach out with any other questions and best of luck with your studies!